6.5 – Flashcards

Unlock all answers in this set

Unlock answers
question
The proper compression-to-ventilation ratio for two-rescuer adult CPR when an oropharyngeal airway is in place is: A. 30:2. B. 5:1. C. asynchronous. D. 15:2.
answer
A. 30:2.
question
When managing cardiac arrest, the appropriate dosing regimen for epinephrine is: A. 1 mg of a 1:10,000 solution every 3 to 5 minutes. B. 10 mL of a 1:1,000 solution every 3 to 5 minutes. C. 0.1 mg/kg of a 1:10,000 solution every 3 minutes. D. 1 mL of a 1:10,000 solution every 3 to 5 minutes.
answer
A. 1 mg of a 1:10,000 solution every 3 to 5 minutes.
question
You respond to the scene of an assault, where a 20-year-old man was struck in the chest with a steel pipe. Your assessment reveals that the patient is unresponsive, apneic, and pulseless. The MOST appropriate next intervention is to: A. look for evidence of a pericardial tamponade. B. immediately assess the patient's cardiac rhythm. C. perform 5 cycles of well-coordinated CPR. D. give 2 minutes of 15 compressions and 2 breaths.
answer
C. perform 5 cycles of well-coordinated CPR.
question
The preferred antiarrhythmic medication and initial dose for a patient with refractory ventricular fibrillation or pulseless ventricular tachycardia is: A. procainamide, 20 mg/min. B. lidocaine, 0.75 mg/kg. C. amiodarone, 300 mg. D. lidocaine, 1.5 mg/kg.
answer
C. amiodarone, 300 mg.
question
Atherosclerosis is a process in which: A. plaque ruptures from a distant location and lodges in one of the coronary arteries. B. plaque infiltrates the arterial wall, decreasing its elasticity and narrowing its lumen. C. the outer wall of a coronary artery becomes lined with masses of fatty tissue. D. calcium precipitates into the arterial walls, greatly reducing the artery's elasticity.
answer
B. plaque infiltrates the arterial wall, decreasing its elasticity and narrowing its lumen.
question
After delivering a shock to a patient in pulseless ventricular tachycardia, you should: A. deliver two effective ventilations. B. check for a pulse. C. reassess the cardiac rhythm. D. resume CPR
answer
D. resume CPR
question
The MOST important initial pieces of equipment to bring to the side of an unresponsive patient are the: A. defibrillator and airway management equipment. B. intubation kit and equipment for vascular access. C. drug kit and stretcher with a long backboard. D. pocket face mask and equipment for intubation.
answer
A. defibrillator and airway management equipment.
question
Once an advanced airway device has been inserted into a cardiac arrest patient: A. you should deliver one breath every 5 to 6 seconds. B. the compressor should pause so ventilations can be given. C. chest compressions should be increased to 120 per minute. D. ventilations are delivered at a rate of 8 to 10 breaths/min.
answer
D. ventilations are delivered at a rate of 8 to 10 breaths/min.
question
When performing CPR on an adult patient in cardiac arrest, it is important to: A. limit interruptions in chest compressions to 20 seconds. B. allow the chest to fully recoil between compressions. C. deliver at least 80 to 90 compressions per minute. D. deliver forceful ventilations between compressions.
answer
B. allow the chest to fully recoil between compressions.
question
You are called to a local gym for a patient with nausea. Your patient, a 29-year-old man, tells you that he thinks he has a "stomach bug." He is conscious and alert, denies chest pain or shortness of breath, and tells you that he has been nauseated for the last 4 hours but has not vomited. His blood pressure is 124/66 mm Hg, pulse is 46 beats/min and strong, respirations are 20 breaths/min and regular, and room air oxygen saturation is 99%. The cardiac monitor reveals a sinus bradycardia. You should: A. give 100% oxygen, start two large-bore IV lines, administer 20 mL/kg normal saline boluses, and transport. B. advise him that he can probably drive himself to the emergency department or schedule an appointment with his physician. C. apply high-flow oxygen via nonrebreathing mask, start an IV line, administer 0.5 mg of atropine, and transport to the closest facility. D. administer oxygen via nasal cannula, start an IV of normal saline, consider administering an antiemetic, and transport.
answer
D. administer oxygen via nasal cannula, start an IV of normal saline, consider administering an antiemetic, and transport.
question
If you are using a biphasic defibrillator, but are unsure of the appropriate starting energy setting, you should set the defibrillator to: A. 120 J. B. 200 J. C. 150 J. D. 360 J.
answer
B. 200 J.
question
Treatment for a patient with bradycardia and significantly compromised cardiac output includes: A. transcutaneous cardiac pacing. B. a dopamine infusion at 20 mg/min. C. 1 mg of atropine via IV push. D. 1 mg of epinephrine 1:10,000.
answer
A. transcutaneous cardiac pacing.
question
You and your partner arrive at the scene of an unresponsive male patient. Your assessment reveals that he is pulseless and apneic. The patient's wife tells you that he collapsed about 10 minutes ago. You should: A. ask the patient's wife if her husband has a living will. B. initiate CPR as your partner applies the defibrillator pads. C. apply the defibrillator pads and assess his cardiac rhythm. D. begin CPR with a compression to ventilation ratio of 15:2.
answer
B. initiate CPR as your partner applies the defibrillator pads.
question
You have applied the cardiac monitor to your 66-year-old male cardiac arrest patient and see what appears to be asystole. You should: A. check for a pulse for a maximum of 10 seconds. B. resume CPR and place an advanced airway device. C. continue CPR and reassess the rhythm in 2 minutes. D. assess another lead or increase the gain sensitivity.
answer
D. assess another lead or increase the gain sensitivity.
question
The recommended first-line treatment for third-degree heart block associated with bradycardia and hemodynamic compromise is: A. a dopamine infusion. B. atropine sulfate. C. transcutaneous pacing. D. an epinephrine infusion.
answer
C. transcutaneous pacing.
question
Common causes of cardiac arrest include all of the following, EXCEPT: A. hyperglycemia. B. cardiac tamponade. C. pulmonary embolism. D. hypovolemia.
answer
A. hyperglycemia.
question
A 56-year-old man presents with an acute onset of chest pressure and diaphoresis. He has a history of hypertension and type 2 diabetes. His airway is patent and his breathing is adequate. You should: A. obtain baseline vital signs. B. administer supplemental oxygen. C. acquire a 12-lead ECG tracing. D. establish vascular access.
answer
B. administer supplemental oxygen.
question
A patient is considered a potential candidate for fibrinolytic therapy if he or she has experienced chest discomfort for: A. more than 5 minutes. B. less than 15 minutes C. more than 24 hours. D. less than 12 hours.
answer
D. less than 12 hours.
question
You receive a call to a skilled nursing facility for an elderly man with generalized weakness. The patient's nurse tells you that he has not eaten anything in the past 18 hours. She presents you with his chart, which states that he has a history of atrial fibrillation, congestive heart failure, and hypertension. His medications include Vasotec, digoxin, and warfarin. As you apply the cardiac monitor, your partner takes the patient's vital signs, which reveal a blood pressure of 136/76 mm Hg, a rapid and irregular pulse, and respirations of 22 breaths/min. The ECG reveals atrial fibrillation with a variable rate between 110 and 130 beats/min, and a 12-lead ECG tracing reveals the same. After applying supplemental oxygen, you should: A. establish IV access, give a beta blocker to slow his heart rate, and transport. B. place him in a position of comfort and transport with continuous monitoring. C. start an IV line set to keep the vein open, administer diltiazem, and transport. D. establish an IV, administer Versed, and perform synchronized cardioversion.
answer
C. start an IV line set to keep the vein open, administer diltiazem, and transport.
question
Regardless of the patient's presenting cardiac arrest rhythm, the first IV or IO drug that should be given is: A. calcium chloride. B. an inotrope. C. a vasopressor. D. an antidysrhythmic.
answer
C. a vasopressor.
question
Which of the following actions should NOT occur while CPR is in progress? A. Advanced airway placement B. Establishment of vascular access C. Assessment for a palpable pulse D. Cardiac rhythm assessment
answer
D. Cardiac rhythm assessment
question
A 70-year-old woman remains in asystole following 10 minutes of well-coordinated CPR, successful intubation, IV therapy, and three doses of epinephrine. There are no obvious underlying causes that would explain her cardiac arrest. At this point, it would be appropriate to: A. defibrillate one time in case she is in V-Fib. B. seriously consider ceasing resuscitative efforts. C. attempt transcutaneous cardiac pacing. D. transport at once with CPR continuing en route.
answer
B. seriously consider ceasing resuscitative efforts.
question
After administering 40 units of vasopressin to an elderly woman in bradycardic pulseless electrical activity, you should: A. pause CPR for no longer than 10 seconds and assess for a pulse. B. administer 1 mg of epinephrine 1:10,000 while CPR is ongoing. C. give naloxone to rule out opiate overdose as the cause of her arrest. D. continue CPR and flush the IV line with 20 mL of normal saline.
answer
D. continue CPR and flush the IV line with 20 mL of normal saline.
question
Which of the following factors would present the GREATEST difficulty when distinguishing supraventricular tachycardia from ventricular tachycardia? A. Absence of P waves B. Aberrant conduction C. Retrograde conduction D. The rate of the rhythm
answer
B. Aberrant conduction
question
Myocardial ischemia occurs when the heart muscle: A. undergoes necrosis because of prolonged oxygen deprivation. B. suffers oxygen deprivation secondary to coronary vasodilation. C. is deprived of oxygen because of a blocked coronary artery. D. experiences a decreased oxygen demand and an increased supply. Incorrect
answer
C. is deprived of oxygen because of a blocked coronary artery.
question
The preferred antiarrhythmic medication and initial dose for a patient with refractory ventricular fibrillation or pulseless ventricular tachycardia is: A. amiodarone, 300 mg. B. lidocaine, 0.75 mg/kg. C. procainamide, 20 mg/min. D. lidocaine, 1.5 mg/kg.
answer
A. amiodarone, 300 mg.
question
You and an EMT are performing CPR on an elderly woman in cardiac arrest as your paramedic partner prepares to intubate her. After the patient has been intubated and proper ET tube placement has been confirmed, you should: A. direct your partner to deliver one breath every 3 to 5 seconds as the EMT-B continues chest compressions. B. administer 2.5 mg of epinephrine via the ET tube and hyperventilate the patient to ensure drug dispersal. C. instruct the EMT-B to pause after 30 compressions so your partner can deliver two ventilations. D. perform asynchronous CPR while ventilating the patient at a rate of 8 to 10 breaths/min.
answer
D. perform asynchronous CPR while ventilating the patient at a rate of 8 to 10 breaths/min.
question
A 33-year-old woman presents with an acute onset of "fluttering" in her chest. She is conscious and alert but is somewhat anxious. She denies any significant medical problems but states that she has been under a lot of stress at work. You apply the cardiac monitor, which reveals a narrow QRS complex tachycardia at a rate of 170 beats/min. The patient's blood pressure is 140/90 mm Hg, and she is breathing without difficulty. The MOST appropriate treatment for this patient involves: A. oxygen, emotional support, and 2.5 mg of midazolam IM. B. oxygen, vagal maneuvers, and emotional support. C. vagal maneuvers, IV access, and 0.25 mg/kg of diltiazem. D. oxygen, IV access, vagal maneuvers, and 6 mg of adenosine.
answer
D. oxygen, IV access, vagal maneuvers, and 6 mg of adenosine.
question
Common causes of cardiac arrest include all of the following, EXCEPT: A. cardiac tamponade. B. hypovolemia. C. hyperglycemia. D. pulmonary embolism.
answer
C. hyperglycemia.
question
Which of the following pulseless rhythms is NOT treated as pulseless electrical activity? A. Ventricular tachycardia B. Idioventricular rhythm C. Junctional escape rhythm D. Sinus bradycardia
answer
A. Ventricular tachycardia
question
You have just performed synchronized cardioversion on a patient with unstable ventricular tachycardia. Upon reassessment, you note that the patient is unresponsive, apneic, and pulseless. You should: A. increase the energy setting on the defibrillator and repeat the cardioversion. B. desynchronize the defibrillator, defibrillate one time, and check for a pulse. C. perform five cycles of CPR, reassess the cardiac rhythm, and defibrillate if needed. D. ensure that the synchronizer is off, defibrillate, and immediately begin CPR.
answer
D. ensure that the synchronizer is off, defibrillate, and immediately begin CPR.
question
When assessing an anxious patient who presents with tachycardia, you must: A. prepare for cardioversion if the rate is less than 150 beats/min. B. administer diazepam or midazolam to facilitate your assessment. C. obtain a 12-lead ECG tracing before initiating any treatment. D. determine if the tachycardia is causing hemodynamic instability.
answer
D. determine if the tachycardia is causing hemodynamic instability.
question
You are performing CPR on an 80-year-old woman whose cardiac arrest was witnessed by her husband. Several intubation attempts have been unsuccessful, but ventilations with a bag-mask device are producing adequate chest rise. IV access has been obtained and 1 mg of epinephrine has been administered. The cardiac monitor displays a narrow QRS complex rhythm at a rate of 70 beats/min. According to the patient's husband, she has had numerous episodes of diarrhea over the past 24 hours and has not had much of an appetite. The MOST appropriate next action should be to: A. assess the rhythm and pulse after 3 minutes of CPR. B. administer 1 mg of atropine while CPR is ongoing. C. continue CPR and administer crystalloid fluid boluses. D. administer 50% dextrose for presumed hypoglycemia.
answer
C. continue CPR and administer crystalloid fluid boluses.
question
You have restored spontaneous circulation in a 54-year-old man who was in ventricular fibrillation. During the arrest interval, you delivered 2 shocks, 1 mg of epinephrine, and 300 mg of amiodarone. The patient's blood pressure is 96/60 mm Hg, and the cardiac monitor displays a sinus rhythm at a rate of 70 beats/min with frequent premature ventricular complexes. Appropriate post-resuscitation care for this patient includes: A. 0.5 mg of atropine sulfate. B. a low-dose dopamine infusion. C. a 20-mL/kg crystalloid bolus. D. an infusion of amiodarone
answer
D. an infusion of amiodarone
question
Following 2 minutes of CPR, you reassess an unresponsive man's pulse and cardiac rhythm. He remains pulseless and the monitor displays coarse ventricular fibrillation. You should: A. resume CPR as the defibrillator is charging. B. perform 2 minutes of CPR and then reassess. C. continue CPR and intubate his trachea. D. continue CPR and establish IV or IO access.
answer
A. resume CPR as the defibrillator is charging.
question
A 39-year-old man in asystole has been unresponsive to high-quality CPR and two doses of epinephrine. The patient is intubated and an IO catheter is in place. You should focus on: A. providing mild hyperventilation. B. searching for reversible causes. C. establishing a peripheral IV line D. transcutaneous cardiac pacing
answer
B. searching for reversible causes.
question
In which of the following situations would you likely NOT be able to palpate a pulse despite effective chest compressions? A. Profound hypoxia B. Severe acidosis C. Tension pneumothorax D. Hyperkalemia
answer
C. Tension pneumothorax
question
When assessing an anxious patient who presents with tachycardia, you must: A. administer diazepam or midazolam to facilitate your assessment. B. prepare for cardioversion if the rate is less than 150 beats/min. C. determine if the tachycardia is causing hemodynamic instability. D. obtain a 12-lead ECG tracing before initiating any treatment.
answer
C. determine if the tachycardia is causing hemodynamic instability.
question
A middle-aged man in ventricular fibrillation has been refractory to several biphasic defibrillations, well-coordinated CPR, adequately performed ventilations, and two doses of epinephrine. What should you do next? A. Administer 300 mg of amiodarone via rapid IV push B. Give amiodarone followed by 1.5 mg/kg of lidocaine C. Rapidly infuse 2 liters of normal saline solution D. Give 40 units of vasopressin followed by defibrillation
answer
A. Administer 300 mg of amiodarone via rapid IV push
question
Fibrinolysis may be contraindicated in all of the following, EXCEPT: A. major trauma or surgery within the past 4 weeks. B. a history of anaphylactic shock caused by salicylates. C. significant closed head trauma within the past 3 weeks. D. a history of structural central nervous system disease.
answer
B. a history of anaphylactic shock caused by salicylates.
question
You have applied the defibrillator pads to a pulseless and apneic 60-year-old woman and observe a slow, wide QRS complex rhythm. Your next action should be to: A. attempt cardiac pacing. B. resume CPR at once. C. check the carotid pulse. D. assess breathing effort.
answer
B. resume CPR at once.
question
A 68-year-old woman presents with an acute onset of confusion, shortness of breath, and diaphoresis. Her blood pressure is 72/50 mm Hg, her heart rate is slow and weak, and her respirations are increased and shallow. The ECG reveals a third-degree heart block at a rate of 38 beats/min. After placing the patient on high-flow oxygen, you should: A. obtain a 12-lead ECG to detect an acute myocardial infarction. B. obtain vascular access and give a fluid bolus. C. immediately attempt transcutaneous pacing. D. start an IV and administer 0.5 mg atropine.
answer
C. immediately attempt transcutaneous pacing.
question
A 41-year-old man complains of chest heaviness and mild shortness of breath that began about 2 hours ago. He is conscious and alert. As you are assessing him, he tells you that he has high blood pressure for which he takes Clonidine. His blood pressure is 160/90 mm Hg, heart rate is 140 beats/min and regular, and respirations are 22 breaths/min and somewhat labored. The cardiac monitor displays a narrow complex tachycardia in lead II. Which of the following interventions is NOT indicated for this patient? A. IV access B. Aspirin C. 12-Lead ECG D. Adenosine
answer
D. Adenosine
question
What is the approximate maximum dose of lidocaine for a 200-pound patient? A. 300 mg B. 275 mg C. 325 mg D. 350 mg
answer
B. 275 mg
question
If a patient remains comatose following return of spontaneous circulation, you should: A. immediately obtain a 12-lead. B. begin an infusion of dopamine. C. begin hypothermia treatment. D. provide mild hyperventilation.
answer
D. provide mild hyperventilation.
question
If a patient remains comatose following return of spontaneous circulation, you should: A. immediately obtain a 12-lead. B. provide mild hyperventilation. C. begin an infusion of dopamine. D. begin hypothermia treatment.
answer
D. begin hypothermia treatment.
Get an explanation on any task
Get unstuck with the help of our AI assistant in seconds
New